Does this property of comaximal ideals always hold?Unital commutative ring and distinct maximal ideals.Intersection of distinct maximal ideals in a commutative ring with identity.Where does the proof for commutative rings break down in the non-commutative ring when showing only two ideals implies the ring is a field?Direct-Sum Decomposition of an Artinian moduleProve that $m_1m_2ldots m_r=n_1n_2ldots n_s$ implies $r=s$ for distinct maximal idealshow is $m_1 m_2…m_i/m_1 m_2…m_i+1$ a vector space over $A/m_i+1$?Question about maximal ideals in a commutative Artinian ringDoes an Artinian ring have only finitely many maximal left ideals?Are non-coprime ideals always contained in some prime ideal?Product of ideals equals intersection but they are not comaximal

Is it possible to have a strip of cold climate in the middle of a planet?

Can someone explain how this makes sense electrically?

Aragorn's "guise" in the Orthanc Stone

Why did the HMS Bounty go back to a time when whales are already rare?

When were female captains banned from Starfleet?

What should you do if you miss a job interview (deliberately)?

Loading commands from file

Is "staff" singular or plural?

250 Floor Tower

Pre-mixing cryogenic fuels and using only one fuel tank

When a Cleric spontaneously casts a Cure Light Wounds spell, will a Pearl of Power recover the original spell or Cure Light Wounds?

Argument list too long when zipping large list of certain files in a folder

Is there a single word describing earning money through any means?

If a character has darkvision, can they see through an area of nonmagical darkness filled with lightly obscuring gas?

How can Trident be so inexpensive? Will it orbit Triton or just do a (slow) flyby?

Why did the EU agree to delay the Brexit deadline?

Calculating Wattage for Resistor in High Frequency Application?

Electoral considerations aside, what are potential benefits, for the US, of policy changes proposed by the tweet recognizing Golan annexation?

Problem with TransformedDistribution

Melting point of aspirin, contradicting sources

What is this called? Old film camera viewer?

Lowest total scrabble score

Should I stop contributing to retirement accounts?

Did arcade monitors have same pixel aspect ratio as TV sets?



Does this property of comaximal ideals always hold?


Unital commutative ring and distinct maximal ideals.Intersection of distinct maximal ideals in a commutative ring with identity.Where does the proof for commutative rings break down in the non-commutative ring when showing only two ideals implies the ring is a field?Direct-Sum Decomposition of an Artinian moduleProve that $m_1m_2ldots m_r=n_1n_2ldots n_s$ implies $r=s$ for distinct maximal idealshow is $m_1 m_2…m_i/m_1 m_2…m_i+1$ a vector space over $A/m_i+1$?Question about maximal ideals in a commutative Artinian ringDoes an Artinian ring have only finitely many maximal left ideals?Are non-coprime ideals always contained in some prime ideal?Product of ideals equals intersection but they are not comaximal













5












$begingroup$


I am reading a paper in which the following result is used, but I can’t see the proof of this.




Let $R$ be a commutative ring with only two maximal ideals, say $M_1$ and $M_2$. Suppose $m_1 in M_1$ is such that $m_1 notin M_2$. Then can we always find $m_2 in M_2$ such that $m_1+m_2=1$?




Any ideas?










share|cite|improve this question











$endgroup$











  • $begingroup$
    Consider the ideal generated by $M_2$ and $m_1$, this ideal must be $R=(1)$ since $M_2$ is maximal
    $endgroup$
    – B.Swan
    Mar 15 at 2:15






  • 1




    $begingroup$
    @B.Swan this approach doesn't work, to see why try writing out the details
    $endgroup$
    – Alex Mathers
    Mar 15 at 2:17






  • 1




    $begingroup$
    Set $I=(M_2 cup m_1) $, the ideal generated by $M_2$ and $m_1$. Elements of $I$ have the form $x+rm_1$, where $x in M_2$ and $r in R$. Since $m_1 notin M_2$ and $M_2$ maximal, it follows $I=R$. Thus there exists $s in R$ with $1=x+sm_1$. And I guess one gets stuck here. Sorry for the wrong approach and thanks for pointing it out.
    $endgroup$
    – B.Swan
    Mar 15 at 2:27
















5












$begingroup$


I am reading a paper in which the following result is used, but I can’t see the proof of this.




Let $R$ be a commutative ring with only two maximal ideals, say $M_1$ and $M_2$. Suppose $m_1 in M_1$ is such that $m_1 notin M_2$. Then can we always find $m_2 in M_2$ such that $m_1+m_2=1$?




Any ideas?










share|cite|improve this question











$endgroup$











  • $begingroup$
    Consider the ideal generated by $M_2$ and $m_1$, this ideal must be $R=(1)$ since $M_2$ is maximal
    $endgroup$
    – B.Swan
    Mar 15 at 2:15






  • 1




    $begingroup$
    @B.Swan this approach doesn't work, to see why try writing out the details
    $endgroup$
    – Alex Mathers
    Mar 15 at 2:17






  • 1




    $begingroup$
    Set $I=(M_2 cup m_1) $, the ideal generated by $M_2$ and $m_1$. Elements of $I$ have the form $x+rm_1$, where $x in M_2$ and $r in R$. Since $m_1 notin M_2$ and $M_2$ maximal, it follows $I=R$. Thus there exists $s in R$ with $1=x+sm_1$. And I guess one gets stuck here. Sorry for the wrong approach and thanks for pointing it out.
    $endgroup$
    – B.Swan
    Mar 15 at 2:27














5












5








5


1



$begingroup$


I am reading a paper in which the following result is used, but I can’t see the proof of this.




Let $R$ be a commutative ring with only two maximal ideals, say $M_1$ and $M_2$. Suppose $m_1 in M_1$ is such that $m_1 notin M_2$. Then can we always find $m_2 in M_2$ such that $m_1+m_2=1$?




Any ideas?










share|cite|improve this question











$endgroup$




I am reading a paper in which the following result is used, but I can’t see the proof of this.




Let $R$ be a commutative ring with only two maximal ideals, say $M_1$ and $M_2$. Suppose $m_1 in M_1$ is such that $m_1 notin M_2$. Then can we always find $m_2 in M_2$ such that $m_1+m_2=1$?




Any ideas?







abstract-algebra ring-theory maximal-and-prime-ideals






share|cite|improve this question















share|cite|improve this question













share|cite|improve this question




share|cite|improve this question








edited Mar 15 at 18:42









Peter Mortensen

563310




563310










asked Mar 15 at 2:05









Math LoverMath Lover

1,034315




1,034315











  • $begingroup$
    Consider the ideal generated by $M_2$ and $m_1$, this ideal must be $R=(1)$ since $M_2$ is maximal
    $endgroup$
    – B.Swan
    Mar 15 at 2:15






  • 1




    $begingroup$
    @B.Swan this approach doesn't work, to see why try writing out the details
    $endgroup$
    – Alex Mathers
    Mar 15 at 2:17






  • 1




    $begingroup$
    Set $I=(M_2 cup m_1) $, the ideal generated by $M_2$ and $m_1$. Elements of $I$ have the form $x+rm_1$, where $x in M_2$ and $r in R$. Since $m_1 notin M_2$ and $M_2$ maximal, it follows $I=R$. Thus there exists $s in R$ with $1=x+sm_1$. And I guess one gets stuck here. Sorry for the wrong approach and thanks for pointing it out.
    $endgroup$
    – B.Swan
    Mar 15 at 2:27

















  • $begingroup$
    Consider the ideal generated by $M_2$ and $m_1$, this ideal must be $R=(1)$ since $M_2$ is maximal
    $endgroup$
    – B.Swan
    Mar 15 at 2:15






  • 1




    $begingroup$
    @B.Swan this approach doesn't work, to see why try writing out the details
    $endgroup$
    – Alex Mathers
    Mar 15 at 2:17






  • 1




    $begingroup$
    Set $I=(M_2 cup m_1) $, the ideal generated by $M_2$ and $m_1$. Elements of $I$ have the form $x+rm_1$, where $x in M_2$ and $r in R$. Since $m_1 notin M_2$ and $M_2$ maximal, it follows $I=R$. Thus there exists $s in R$ with $1=x+sm_1$. And I guess one gets stuck here. Sorry for the wrong approach and thanks for pointing it out.
    $endgroup$
    – B.Swan
    Mar 15 at 2:27
















$begingroup$
Consider the ideal generated by $M_2$ and $m_1$, this ideal must be $R=(1)$ since $M_2$ is maximal
$endgroup$
– B.Swan
Mar 15 at 2:15




$begingroup$
Consider the ideal generated by $M_2$ and $m_1$, this ideal must be $R=(1)$ since $M_2$ is maximal
$endgroup$
– B.Swan
Mar 15 at 2:15




1




1




$begingroup$
@B.Swan this approach doesn't work, to see why try writing out the details
$endgroup$
– Alex Mathers
Mar 15 at 2:17




$begingroup$
@B.Swan this approach doesn't work, to see why try writing out the details
$endgroup$
– Alex Mathers
Mar 15 at 2:17




1




1




$begingroup$
Set $I=(M_2 cup m_1) $, the ideal generated by $M_2$ and $m_1$. Elements of $I$ have the form $x+rm_1$, where $x in M_2$ and $r in R$. Since $m_1 notin M_2$ and $M_2$ maximal, it follows $I=R$. Thus there exists $s in R$ with $1=x+sm_1$. And I guess one gets stuck here. Sorry for the wrong approach and thanks for pointing it out.
$endgroup$
– B.Swan
Mar 15 at 2:27





$begingroup$
Set $I=(M_2 cup m_1) $, the ideal generated by $M_2$ and $m_1$. Elements of $I$ have the form $x+rm_1$, where $x in M_2$ and $r in R$. Since $m_1 notin M_2$ and $M_2$ maximal, it follows $I=R$. Thus there exists $s in R$ with $1=x+sm_1$. And I guess one gets stuck here. Sorry for the wrong approach and thanks for pointing it out.
$endgroup$
– B.Swan
Mar 15 at 2:27











2 Answers
2






active

oldest

votes


















8












$begingroup$

Take $R=mathbbQtimesmathbbQ$, $M_1=mathbbQtimes0$, $M_2=0timesmathbbQ$, and $m_1=(2,0)in M_1setminus M_2$. Then $(1,1)inmathbbQtimesmathbbQ$ satisfies that $$(1,1)-(2,0)=(-1,1)notin M_2$$



Therefore, that property is not satisfied in general.



Maybe the property that they are really using is that there exist $ain M_1$ and $bin M_2$ such that $a+b=1$. Not arbitrary $a,b$. This other property is immediate by using the maximality of $M_1$ and $M_2$, which implies that $M_1+M_2=R$.






share|cite|improve this answer











$endgroup$




















    5












    $begingroup$

    First notice that $1-m_1$ cannot be a unit, because this would imply $m_1$ is in the Jacobson radical of $R$, and in particular we would have $m_1in M_2$.



    Now it follows that the ideal of $R$ generated by $1-m_1$ must be contained in a maximal ideal, but it cannot be contained in $M_1$ because then it would follow that $1in M_1$. Thus this ideal is contained in $M_2$ (the only other maximal ideal), i.e. you get $1-m_1in M_2$.




    Edit: I think my reasoning for $1-m_1$ not being a unit is wrong (it seems we would need that $1-m_1x$ is a unit for every $xin R$ to conclude $m_1$ is in the Jacobson radical). The rest of the argument goes through, so I'm going to leave my answer up for a while in hopes that somebody can help figure that part out.






    share|cite|improve this answer











    $endgroup$












      Your Answer





      StackExchange.ifUsing("editor", function ()
      return StackExchange.using("mathjaxEditing", function ()
      StackExchange.MarkdownEditor.creationCallbacks.add(function (editor, postfix)
      StackExchange.mathjaxEditing.prepareWmdForMathJax(editor, postfix, [["$", "$"], ["\\(","\\)"]]);
      );
      );
      , "mathjax-editing");

      StackExchange.ready(function()
      var channelOptions =
      tags: "".split(" "),
      id: "69"
      ;
      initTagRenderer("".split(" "), "".split(" "), channelOptions);

      StackExchange.using("externalEditor", function()
      // Have to fire editor after snippets, if snippets enabled
      if (StackExchange.settings.snippets.snippetsEnabled)
      StackExchange.using("snippets", function()
      createEditor();
      );

      else
      createEditor();

      );

      function createEditor()
      StackExchange.prepareEditor(
      heartbeatType: 'answer',
      autoActivateHeartbeat: false,
      convertImagesToLinks: true,
      noModals: true,
      showLowRepImageUploadWarning: true,
      reputationToPostImages: 10,
      bindNavPrevention: true,
      postfix: "",
      imageUploader:
      brandingHtml: "Powered by u003ca class="icon-imgur-white" href="https://imgur.com/"u003eu003c/au003e",
      contentPolicyHtml: "User contributions licensed under u003ca href="https://creativecommons.org/licenses/by-sa/3.0/"u003ecc by-sa 3.0 with attribution requiredu003c/au003e u003ca href="https://stackoverflow.com/legal/content-policy"u003e(content policy)u003c/au003e",
      allowUrls: true
      ,
      noCode: true, onDemand: true,
      discardSelector: ".discard-answer"
      ,immediatelyShowMarkdownHelp:true
      );



      );













      draft saved

      draft discarded


















      StackExchange.ready(
      function ()
      StackExchange.openid.initPostLogin('.new-post-login', 'https%3a%2f%2fmath.stackexchange.com%2fquestions%2f3148803%2fdoes-this-property-of-comaximal-ideals-always-hold%23new-answer', 'question_page');

      );

      Post as a guest















      Required, but never shown

























      2 Answers
      2






      active

      oldest

      votes








      2 Answers
      2






      active

      oldest

      votes









      active

      oldest

      votes






      active

      oldest

      votes









      8












      $begingroup$

      Take $R=mathbbQtimesmathbbQ$, $M_1=mathbbQtimes0$, $M_2=0timesmathbbQ$, and $m_1=(2,0)in M_1setminus M_2$. Then $(1,1)inmathbbQtimesmathbbQ$ satisfies that $$(1,1)-(2,0)=(-1,1)notin M_2$$



      Therefore, that property is not satisfied in general.



      Maybe the property that they are really using is that there exist $ain M_1$ and $bin M_2$ such that $a+b=1$. Not arbitrary $a,b$. This other property is immediate by using the maximality of $M_1$ and $M_2$, which implies that $M_1+M_2=R$.






      share|cite|improve this answer











      $endgroup$

















        8












        $begingroup$

        Take $R=mathbbQtimesmathbbQ$, $M_1=mathbbQtimes0$, $M_2=0timesmathbbQ$, and $m_1=(2,0)in M_1setminus M_2$. Then $(1,1)inmathbbQtimesmathbbQ$ satisfies that $$(1,1)-(2,0)=(-1,1)notin M_2$$



        Therefore, that property is not satisfied in general.



        Maybe the property that they are really using is that there exist $ain M_1$ and $bin M_2$ such that $a+b=1$. Not arbitrary $a,b$. This other property is immediate by using the maximality of $M_1$ and $M_2$, which implies that $M_1+M_2=R$.






        share|cite|improve this answer











        $endgroup$















          8












          8








          8





          $begingroup$

          Take $R=mathbbQtimesmathbbQ$, $M_1=mathbbQtimes0$, $M_2=0timesmathbbQ$, and $m_1=(2,0)in M_1setminus M_2$. Then $(1,1)inmathbbQtimesmathbbQ$ satisfies that $$(1,1)-(2,0)=(-1,1)notin M_2$$



          Therefore, that property is not satisfied in general.



          Maybe the property that they are really using is that there exist $ain M_1$ and $bin M_2$ such that $a+b=1$. Not arbitrary $a,b$. This other property is immediate by using the maximality of $M_1$ and $M_2$, which implies that $M_1+M_2=R$.






          share|cite|improve this answer











          $endgroup$



          Take $R=mathbbQtimesmathbbQ$, $M_1=mathbbQtimes0$, $M_2=0timesmathbbQ$, and $m_1=(2,0)in M_1setminus M_2$. Then $(1,1)inmathbbQtimesmathbbQ$ satisfies that $$(1,1)-(2,0)=(-1,1)notin M_2$$



          Therefore, that property is not satisfied in general.



          Maybe the property that they are really using is that there exist $ain M_1$ and $bin M_2$ such that $a+b=1$. Not arbitrary $a,b$. This other property is immediate by using the maximality of $M_1$ and $M_2$, which implies that $M_1+M_2=R$.







          share|cite|improve this answer














          share|cite|improve this answer



          share|cite|improve this answer








          edited Mar 15 at 3:52

























          answered Mar 15 at 3:00









          user647486user647486

          47919




          47919





















              5












              $begingroup$

              First notice that $1-m_1$ cannot be a unit, because this would imply $m_1$ is in the Jacobson radical of $R$, and in particular we would have $m_1in M_2$.



              Now it follows that the ideal of $R$ generated by $1-m_1$ must be contained in a maximal ideal, but it cannot be contained in $M_1$ because then it would follow that $1in M_1$. Thus this ideal is contained in $M_2$ (the only other maximal ideal), i.e. you get $1-m_1in M_2$.




              Edit: I think my reasoning for $1-m_1$ not being a unit is wrong (it seems we would need that $1-m_1x$ is a unit for every $xin R$ to conclude $m_1$ is in the Jacobson radical). The rest of the argument goes through, so I'm going to leave my answer up for a while in hopes that somebody can help figure that part out.






              share|cite|improve this answer











              $endgroup$

















                5












                $begingroup$

                First notice that $1-m_1$ cannot be a unit, because this would imply $m_1$ is in the Jacobson radical of $R$, and in particular we would have $m_1in M_2$.



                Now it follows that the ideal of $R$ generated by $1-m_1$ must be contained in a maximal ideal, but it cannot be contained in $M_1$ because then it would follow that $1in M_1$. Thus this ideal is contained in $M_2$ (the only other maximal ideal), i.e. you get $1-m_1in M_2$.




                Edit: I think my reasoning for $1-m_1$ not being a unit is wrong (it seems we would need that $1-m_1x$ is a unit for every $xin R$ to conclude $m_1$ is in the Jacobson radical). The rest of the argument goes through, so I'm going to leave my answer up for a while in hopes that somebody can help figure that part out.






                share|cite|improve this answer











                $endgroup$















                  5












                  5








                  5





                  $begingroup$

                  First notice that $1-m_1$ cannot be a unit, because this would imply $m_1$ is in the Jacobson radical of $R$, and in particular we would have $m_1in M_2$.



                  Now it follows that the ideal of $R$ generated by $1-m_1$ must be contained in a maximal ideal, but it cannot be contained in $M_1$ because then it would follow that $1in M_1$. Thus this ideal is contained in $M_2$ (the only other maximal ideal), i.e. you get $1-m_1in M_2$.




                  Edit: I think my reasoning for $1-m_1$ not being a unit is wrong (it seems we would need that $1-m_1x$ is a unit for every $xin R$ to conclude $m_1$ is in the Jacobson radical). The rest of the argument goes through, so I'm going to leave my answer up for a while in hopes that somebody can help figure that part out.






                  share|cite|improve this answer











                  $endgroup$



                  First notice that $1-m_1$ cannot be a unit, because this would imply $m_1$ is in the Jacobson radical of $R$, and in particular we would have $m_1in M_2$.



                  Now it follows that the ideal of $R$ generated by $1-m_1$ must be contained in a maximal ideal, but it cannot be contained in $M_1$ because then it would follow that $1in M_1$. Thus this ideal is contained in $M_2$ (the only other maximal ideal), i.e. you get $1-m_1in M_2$.




                  Edit: I think my reasoning for $1-m_1$ not being a unit is wrong (it seems we would need that $1-m_1x$ is a unit for every $xin R$ to conclude $m_1$ is in the Jacobson radical). The rest of the argument goes through, so I'm going to leave my answer up for a while in hopes that somebody can help figure that part out.







                  share|cite|improve this answer














                  share|cite|improve this answer



                  share|cite|improve this answer








                  edited Mar 15 at 2:29

























                  answered Mar 15 at 2:15









                  Alex MathersAlex Mathers

                  11.1k21344




                  11.1k21344



























                      draft saved

                      draft discarded
















































                      Thanks for contributing an answer to Mathematics Stack Exchange!


                      • Please be sure to answer the question. Provide details and share your research!

                      But avoid


                      • Asking for help, clarification, or responding to other answers.

                      • Making statements based on opinion; back them up with references or personal experience.

                      Use MathJax to format equations. MathJax reference.


                      To learn more, see our tips on writing great answers.




                      draft saved


                      draft discarded














                      StackExchange.ready(
                      function ()
                      StackExchange.openid.initPostLogin('.new-post-login', 'https%3a%2f%2fmath.stackexchange.com%2fquestions%2f3148803%2fdoes-this-property-of-comaximal-ideals-always-hold%23new-answer', 'question_page');

                      );

                      Post as a guest















                      Required, but never shown





















































                      Required, but never shown














                      Required, but never shown












                      Required, but never shown







                      Required, but never shown

































                      Required, but never shown














                      Required, but never shown












                      Required, but never shown







                      Required, but never shown







                      Popular posts from this blog

                      Lowndes Grove History Architecture References Navigation menu32°48′6″N 79°57′58″W / 32.80167°N 79.96611°W / 32.80167; -79.9661132°48′6″N 79°57′58″W / 32.80167°N 79.96611°W / 32.80167; -79.9661178002500"National Register Information System"Historic houses of South Carolina"Lowndes Grove""+32° 48' 6.00", −79° 57' 58.00""Lowndes Grove, Charleston County (260 St. Margaret St., Charleston)""Lowndes Grove"The Charleston ExpositionIt Happened in South Carolina"Lowndes Grove (House), Saint Margaret Street & Sixth Avenue, Charleston, Charleston County, SC(Photographs)"Plantations of the Carolina Low Countrye

                      random experiment with two different functions on unit interval Announcing the arrival of Valued Associate #679: Cesar Manara Planned maintenance scheduled April 23, 2019 at 00:00UTC (8:00pm US/Eastern)Random variable and probability space notionsRandom Walk with EdgesFinding functions where the increase over a random interval is Poisson distributedNumber of days until dayCan an observed event in fact be of zero probability?Unit random processmodels of coins and uniform distributionHow to get the number of successes given $n$ trials , probability $P$ and a random variable $X$Absorbing Markov chain in a computer. Is “almost every” turned into always convergence in computer executions?Stopped random walk is not uniformly integrable

                      How should I support this large drywall patch? Planned maintenance scheduled April 23, 2019 at 00:00UTC (8:00pm US/Eastern) Announcing the arrival of Valued Associate #679: Cesar Manara Unicorn Meta Zoo #1: Why another podcast?How do I cover large gaps in drywall?How do I keep drywall around a patch from crumbling?Can I glue a second layer of drywall?How to patch long strip on drywall?Large drywall patch: how to avoid bulging seams?Drywall Mesh Patch vs. Bulge? To remove or not to remove?How to fix this drywall job?Prep drywall before backsplashWhat's the best way to fix this horrible drywall patch job?Drywall patching using 3M Patch Plus Primer